Difference between revisions of "1970 AHSME Problems/Problem 10"

m (Problem)
 
(One intermediate revision by one other user not shown)
Line 10: Line 10:
  
 
== Solution ==
 
== Solution ==
<math>\fbox{A}</math>
+
Multiplying by <math>100</math> gives <math>100F = 48.181818...</math>.  Subtracting the first equation from the second gives <math>99F = 47.7</math>, and all the other repeating parts cancel out.  This gives <math>F = \frac{47.7}{99} = \frac{477}{990} = \frac{159}{330} = \frac{53}{110}</math>.  Subtracting the numerator from the denominator gives <math>\fbox{D}</math>.
  
 
== See also ==
 
== See also ==
{{AHSME box|year=1970|num-b=9|num-a=11}}   
+
{{AHSME 35p box|year=1970|num-b=9|num-a=11}}   
  
 
[[Category: Introductory Algebra Problems]]
 
[[Category: Introductory Algebra Problems]]
 
{{MAA Notice}}
 
{{MAA Notice}}

Latest revision as of 21:21, 13 July 2019

Problem

Let $F=.48181\cdots$ be an infinite repeating decimal with the digits $8$ and $1$ repeating. When $F$ is written as a fraction in lowest terms, the denominator exceeds the numerator by

$\text{(A) } 13\quad \text{(B) } 14\quad \text{(C) } 29\quad \text{(D) } 57\quad \text{(E) } 126$

Solution

Multiplying by $100$ gives $100F = 48.181818...$. Subtracting the first equation from the second gives $99F = 47.7$, and all the other repeating parts cancel out. This gives $F = \frac{47.7}{99} = \frac{477}{990} = \frac{159}{330} = \frac{53}{110}$. Subtracting the numerator from the denominator gives $\fbox{D}$.

See also

1970 AHSC (ProblemsAnswer KeyResources)
Preceded by
Problem 9
Followed by
Problem 11
1 2 3 4 5 6 7 8 9 10 11 12 13 14 15 16 17 18 19 20 21 22 23 24 25 26 27 28 29 30 31 32 33 34 35
All AHSME Problems and Solutions

The problems on this page are copyrighted by the Mathematical Association of America's American Mathematics Competitions. AMC logo.png